Let $a,b,c,dinBbb{R}$ such that $|c|ne|d|$. Prove that $frac{a+b}{c+d}lefrac{|a|+|b|}{||c|-|d||}$ [closed]












0












$begingroup$


I can figure that $a+ble|a+b|le|a|+|b|$ but I do not know how to deal with $||c|-|d||$










share|cite|improve this question











$endgroup$



closed as off-topic by Carl Mummert, RRL, Cesareo, rtybase, Rebellos Dec 3 '18 at 14:02


This question appears to be off-topic. The users who voted to close gave this specific reason:


  • "This question is missing context or other details: Please improve the question by providing additional context, which ideally includes your thoughts on the problem and any attempts you have made to solve it. This information helps others identify where you have difficulties and helps them write answers appropriate to your experience level." – Carl Mummert, RRL, Cesareo, rtybase, Rebellos

If this question can be reworded to fit the rules in the help center, please edit the question.
















  • $begingroup$
    This question is missing context or other details: Please improve the question by providing additional context, which ideally includes the source and motivation of the problem, your thoughts on the problem and any attempts you have made to solve it. This information helps others identify where you have difficulties and helps them write answers appropriate to your experience level.
    $endgroup$
    – Carl Mummert
    Dec 3 '18 at 2:24
















0












$begingroup$


I can figure that $a+ble|a+b|le|a|+|b|$ but I do not know how to deal with $||c|-|d||$










share|cite|improve this question











$endgroup$



closed as off-topic by Carl Mummert, RRL, Cesareo, rtybase, Rebellos Dec 3 '18 at 14:02


This question appears to be off-topic. The users who voted to close gave this specific reason:


  • "This question is missing context or other details: Please improve the question by providing additional context, which ideally includes your thoughts on the problem and any attempts you have made to solve it. This information helps others identify where you have difficulties and helps them write answers appropriate to your experience level." – Carl Mummert, RRL, Cesareo, rtybase, Rebellos

If this question can be reworded to fit the rules in the help center, please edit the question.
















  • $begingroup$
    This question is missing context or other details: Please improve the question by providing additional context, which ideally includes the source and motivation of the problem, your thoughts on the problem and any attempts you have made to solve it. This information helps others identify where you have difficulties and helps them write answers appropriate to your experience level.
    $endgroup$
    – Carl Mummert
    Dec 3 '18 at 2:24














0












0








0





$begingroup$


I can figure that $a+ble|a+b|le|a|+|b|$ but I do not know how to deal with $||c|-|d||$










share|cite|improve this question











$endgroup$




I can figure that $a+ble|a+b|le|a|+|b|$ but I do not know how to deal with $||c|-|d||$







real-analysis inequality real-numbers absolute-value






share|cite|improve this question















share|cite|improve this question













share|cite|improve this question




share|cite|improve this question








edited Dec 3 '18 at 2:20







Albert Diaz

















asked Dec 3 '18 at 2:06









Albert DiazAlbert Diaz

975




975




closed as off-topic by Carl Mummert, RRL, Cesareo, rtybase, Rebellos Dec 3 '18 at 14:02


This question appears to be off-topic. The users who voted to close gave this specific reason:


  • "This question is missing context or other details: Please improve the question by providing additional context, which ideally includes your thoughts on the problem and any attempts you have made to solve it. This information helps others identify where you have difficulties and helps them write answers appropriate to your experience level." – Carl Mummert, RRL, Cesareo, rtybase, Rebellos

If this question can be reworded to fit the rules in the help center, please edit the question.







closed as off-topic by Carl Mummert, RRL, Cesareo, rtybase, Rebellos Dec 3 '18 at 14:02


This question appears to be off-topic. The users who voted to close gave this specific reason:


  • "This question is missing context or other details: Please improve the question by providing additional context, which ideally includes your thoughts on the problem and any attempts you have made to solve it. This information helps others identify where you have difficulties and helps them write answers appropriate to your experience level." – Carl Mummert, RRL, Cesareo, rtybase, Rebellos

If this question can be reworded to fit the rules in the help center, please edit the question.












  • $begingroup$
    This question is missing context or other details: Please improve the question by providing additional context, which ideally includes the source and motivation of the problem, your thoughts on the problem and any attempts you have made to solve it. This information helps others identify where you have difficulties and helps them write answers appropriate to your experience level.
    $endgroup$
    – Carl Mummert
    Dec 3 '18 at 2:24


















  • $begingroup$
    This question is missing context or other details: Please improve the question by providing additional context, which ideally includes the source and motivation of the problem, your thoughts on the problem and any attempts you have made to solve it. This information helps others identify where you have difficulties and helps them write answers appropriate to your experience level.
    $endgroup$
    – Carl Mummert
    Dec 3 '18 at 2:24
















$begingroup$
This question is missing context or other details: Please improve the question by providing additional context, which ideally includes the source and motivation of the problem, your thoughts on the problem and any attempts you have made to solve it. This information helps others identify where you have difficulties and helps them write answers appropriate to your experience level.
$endgroup$
– Carl Mummert
Dec 3 '18 at 2:24




$begingroup$
This question is missing context or other details: Please improve the question by providing additional context, which ideally includes the source and motivation of the problem, your thoughts on the problem and any attempts you have made to solve it. This information helps others identify where you have difficulties and helps them write answers appropriate to your experience level.
$endgroup$
– Carl Mummert
Dec 3 '18 at 2:24










1 Answer
1






active

oldest

votes


















2












$begingroup$

It is the reversed triangle inequality. For $x,y in mathbb{R}$, we have that $||x| -|y|| leq |x - y|$.



See this post:



Reverse Triangle Inequality Proof






share|cite|improve this answer









$endgroup$




















    1 Answer
    1






    active

    oldest

    votes








    1 Answer
    1






    active

    oldest

    votes









    active

    oldest

    votes






    active

    oldest

    votes









    2












    $begingroup$

    It is the reversed triangle inequality. For $x,y in mathbb{R}$, we have that $||x| -|y|| leq |x - y|$.



    See this post:



    Reverse Triangle Inequality Proof






    share|cite|improve this answer









    $endgroup$


















      2












      $begingroup$

      It is the reversed triangle inequality. For $x,y in mathbb{R}$, we have that $||x| -|y|| leq |x - y|$.



      See this post:



      Reverse Triangle Inequality Proof






      share|cite|improve this answer









      $endgroup$
















        2












        2








        2





        $begingroup$

        It is the reversed triangle inequality. For $x,y in mathbb{R}$, we have that $||x| -|y|| leq |x - y|$.



        See this post:



        Reverse Triangle Inequality Proof






        share|cite|improve this answer









        $endgroup$



        It is the reversed triangle inequality. For $x,y in mathbb{R}$, we have that $||x| -|y|| leq |x - y|$.



        See this post:



        Reverse Triangle Inequality Proof







        share|cite|improve this answer












        share|cite|improve this answer



        share|cite|improve this answer










        answered Dec 3 '18 at 2:09









        riri92riri92

        2118




        2118















            Popular posts from this blog

            Biblatex bibliography style without URLs when DOI exists (in Overleaf with Zotero bibliography)

            ComboBox Display Member on multiple fields

            Is it possible to collect Nectar points via Trainline?